Sufficient condition for bounded Fourier transform

In summary, the author is trying to prove that if F is not bounded and has compact support, then the Fourier transform is infinity. However, this proof relies on the Fourier shift theorem and the compact support assumption, which are both not always valid.
  • #1
mnb96
715
5
Hello,

Let's suppose we are given a function [itex]f:\mathbb{R}\rightarrow \mathbb{R}[/itex], and we assume its Fourier transform [itex]F=\mathcal{F}(f)[/itex] exists and has compact support.

What sufficient condition could we impose on f, in order to be sure that F is also bounded?
 
Physics news on Phys.org
  • #2
f is L1 is certainly sufficient, even if F does not have compact support.
 
  • #3
Thanks a lot!
I think I figured out a way to prove it, but it relies on the Fourier shift theorem and the compact support assumption.
The proof I sketched out is more or less like this:

- Let's suppose F is not bounded and has compact support
- Then there must be at least one point in the support of F where the Fourier transform goes to infinity.
- Let u be that point, and translate the Fourier transform so that u coincides with the origin.
- This means that the inverse FT of the translated F must have integral=infinity.
- But from the Fourier shift-theorem we know that such a function is simply f itself multplied by the complex sinusoid e-iux
-
Note that [itex]|\int_{-\infty}^{+\infty}f(x)e^{-iux}dx| \leq \int_{-\infty}^{+\infty}|f(x)|dx[/itex] which means that f is not absolute integrable.
- Now if we assumed f was in L1 we would reach a contradiction

However I don't see how to generalize this argument to the case when F does not have compact support, e.g. F(u) might go to infinity only for u-->infinity which would render the "translation trick" useless.
 
  • #4
I don't understand your argument. If f is in L1 then, as you noted, the Fourier transform is bounded at every point by the integral of |f(x)|.
 
  • #5
Yes. I have just realized that I in my proof I don't need the statement "there must be at least one point in the support of F where the Fourier transform goes to infinity". The inequality that I wrote simply says that the magnitude of the F at each point is bounded by the absolute integral of f.
Thanks again for clarifying this.
 
  • #6
mnb96 said:
Thanks a lot!
I think I figured out a way to prove it, but it relies on the Fourier shift theorem and the compact support assumption.
The proof I sketched out is more or less like this:

- Let's suppose F is not bounded and has compact support
- Then there must be at least one point in the support of F where the Fourier transform goes to infinity.
- Let u be that point, and translate the Fourier transform so that u coincides with the origin.
- This means that the inverse FT of the translated F must have integral=infinity.
- But from the Fourier shift-theorem we know that such a function is simply f itself multplied by the complex sinusoid e-iux
-
Note that [itex]|\int_{-\infty}^{+\infty}f(x)e^{-iux}dx| \leq \int_{-\infty}^{+\infty}|f(x)|dx[/itex] which means that f is not absolute integrable.
- Now if we assumed f was in L1 we would reach a contradiction

However I don't see how to generalize this argument to the case when F does not have compact support, e.g. F(u) might go to infinity only for u-->infinity which would render the "translation trick" useless.

Cant we use a continuity argument together with compactness to argue boundedness of the transform?
 
  • #7
mnb96 said:
Yes. I have just realized that I in my proof I don't need the statement "there must be at least one point in the support of F where the Fourier transform goes to infinity". The inequality that I wrote simply says that the magnitude of the F at each point is bounded by the absolute integral of f.
Thanks again for clarifying this.
If you look at the reverse question. F bounded with compact support implies f is L1.
 

1. What is a sufficient condition for a bounded Fourier transform?

A sufficient condition for a bounded Fourier transform is that the function must be integrable.

2. How is the boundedness of a Fourier transform related to the integrability of a function?

The boundedness of a Fourier transform is directly related to the integrability of a function because the integral of a function determines the size of its Fourier transform.

3. Can a function with a bounded Fourier transform still have unbounded values?

Yes, a function can have a bounded Fourier transform even if it has unbounded values. The boundedness of the Fourier transform refers to the integral of the function, not the individual values.

4. What is the significance of a bounded Fourier transform in signal processing?

A bounded Fourier transform is significant in signal processing because it ensures that the signal can be reconstructed accurately from its Fourier transform, which is important in applications such as filtering and compression.

5. Is a bounded Fourier transform a necessary condition for a function to have a Fourier transform?

No, a bounded Fourier transform is not a necessary condition for a function to have a Fourier transform. A function can have a Fourier transform even if it is not integrable, but it may not have some properties, such as continuity or differentiability, that are desirable in signal processing applications.

Similar threads

  • Calculus
Replies
3
Views
1K
Replies
2
Views
760
Replies
3
Views
993
  • Calculus
Replies
1
Views
74
  • Calculus
Replies
4
Views
1K
  • Differential Equations
Replies
4
Views
2K
  • Calculus
Replies
13
Views
1K
Replies
4
Views
2K
Back
Top